Chap 4 - CVP Analysis

February 13, 2018 | Author: zyra liam styles | Category: Economic Institutions, Corporate Jargon, Pricing, Economics, Microeconomics
Share Embed Donate


Short Description

Download Chap 4 - CVP Analysis...

Description

Chapter 4

Marginal costing and CVP analysis

90

MULTIPLE CHOICE Basic concepts 1. Cost-volume-profit analysis assumes that over the relevant range A. Variable costs are nonlinear. C. Selling prices are unchanged. B. Fixed costs are nonlinear. D. Total costs are unchanged.

(aicpa)

1. C ? A basic assumption within the relevant range.  Relevant range is a band (or stretch) of activity where the behavior of sales and costs is predictable. The basic assumptions used in management accounting within the relevant range are: a. Costs behavior - costs are segregated as to fixed or variable in behavior. b. Linearity - the behavior of total fixed cost, total variable cost, total costs, and total sales is linear (i.e., straight line). c. Fixed costs - total fixed cost does not change, while unit fixed cost changes; that is, UFC increases as production decreases and it decreases as production increases. d. Variable costs - total variable cost changes in direct relation to change in the volume of production and sales, while unit variable cost is constant. Also, total variable costs are not affected by the change in unit sales price. e. Unit sales price - unit sales price is constant. f. Sales mix - the company produces only one product, or in multi-product operations, the sales mix is constant. g. WIP - there is no work-in-process inventory. h. Production equals sales - there is no change in the number of units in finished goods inventory; meaning, production equals sales. Choice-letter “c” is correct, sales price is assumed to be constant within the relevant range. Choice-letters “a” and “b” are incorrect because variable costs and fixed costs are assumed to be linear. Choice-letter “d” is incorrect because total costs change on account of the total variable costs. 2. Cost-volume-profit analysis assumes that over the relevant range total. A. Revenues are linear. C. Variable costs are nonlinear. B. Costs are unchanged. D. Fixed costs are nonlinear.

(aicpa)

2. A ? Basic assumptions within the relevant range.  Choice-letter “a” is correct because the linearity of revenues and costs is one of the basic assumptions in the CVP analysis. Choice-letter “b” is incorrect because total costs would change because of the change in variable costs. Choice-letters “c” and “d” are incorrect because variable costs and fixed costs are assumed to be linear. 3. Breakeven analysis assumes linearity over the relevant range with respect to Total costs Total revenue Total costs Total revenue A. Yes No C. No Yes

Chapter 4 B.

Marginal costing and CVP analysis Yes (aicpa)

Yes

D.

No

91 No

3. B ? Breakeven linearity assumptions over the relevant range.  Relevant range is a band or width of activity (e.g., in terms of levels of output or the unit of measurement used) where the sales and costs could be predicted with reasonable certainty. Within this range, sales and total costs are assumed to behave linearly. Unit sales price is constant and total sales increase in direct relation with changes in production levels. Total cost is composed of fixed cost and variable cost where total fixed cost is constant regardless of changes in the levels of production and sales and unit variable cost is constant. Total variable costs change in direct relation with the changes in the unit of production levels. Choice-letter “b” is correct because total costs and total revenue are assumed to be linear within the relevant range. 4. Break-even analysis assumes that over the relevant range. A. Selling prices are unchanged. C. Total costs are unchanged. B. Variable costs are nonlinear. D. Fixed costs are nonlinear.

(rpcpa)

4. A ? An assumption about breakeven analysis over the relevant range.  Within the relevant range, the following are the assumptions on cost-volume-profit analysis and breakeven analysis: the behavior of sales and costs is linear, total fixed costs is constant but unit fixed costs changes, total variable costs change but unit variable cost is constant; units sales price is constant; there is no change in the finished goods inventory (i.e., production equals sales); there is no work-in-process inventory; and the sales mix is constant. Within the relevant range, variable cost and fixed costs are linear and total costs change due to the change in variable costs. (Choice-letter “a” is correct).

A. B. C. D.

5. The amount of variable cost per unit and total fixed cost within a relevant range behave this way in relation to production level: Production increases, unit variable cost increases, total fixed cost increases. Production decreases, unit variable cost decreases, total fixed cost decreases. Production increases, unit variable cost remains constant, total fixed cost remains the same. Production increases, unit variable cost decreases, total fixed cost remains the same. (rpcpa) 5. C ? The behavior of variable cost per unit and total fixed costs within the relevant range.  Total fixed costs remain constant within the relevant range. However, unit fixed cost decreases as production increases, and it increases as production decreases. Total variable cost changes in direct relation to the change in the level of production and sales within the relevant range. However, unit variable cost is constant within the relevant range.

Chapter 4

Marginal costing and CVP analysis

92

Therefore, regardless of the change in the level of production, the total fixed costs and unit variable will be constant within the relevant range. Choice-letter “c” is the correct answer. 6. Assuming that a flexible budget is in use, production levels are expected to increase within a relevant ranged, the expected effect on fixed cost per unit (FCU) and variable costs per unit (VCU) would be A. FCU to decrease and VCU to decrease. B. FCU to decrease and VCU no change. C. FCU no change and VCU no change. D. FCU no change and VCU to decrease. (rpcpa) 6. B ? Using the flexible budget and assuming production increases within a relevant range, determine the expected effect on fixed costs per unit (FCU) and variable cost per unit (VCU) by an increase in production levels.  Within the relevant range, total fixed cost remains constant but FCU changes (that is, FCU decreases as production increases and FCU increases as production decreases). Also within the relevant range, total variable costs changes but VCU is constant. Choice-letter “b” is correct, FCU decreases, VCU does not change. Choice-letter “b” is correct. 7. One of the major assumptions limiting to reliability of break-even analysis is that A. The cost of productivity will continually increase. B. The cost of production factors varies with changes in technology. C. Total variable cost will remain unchanged over the relevant range. D. Total fixed cost will remain unchanged over the relevant range. (rpcpa) 7. D ? A major assumption limiting the reliability of breakeven analysis.  The breakeven analysis is based on the following assumptions within the relevant range: a. The behavior of sales and costs is linear within the relevant range. b. Total fixed costs are constant but unit fixed cost changes. c. Total variable costs change but unit variable cost is constant. d. Unit sales price is constant. e. Sales equal production; there is no change in the finished goods inventory. f. There is no work-in-process inventory. g. The sales mix ratio is constant. Choice-letter “d” is correct (refer to assumption letter ”b”), Choice-letter “a” is incorrect because efficiency and productivity have no direct relations with the BEP analysis; choice-letter “b” is incorrect because it is not an assumption in the BEP analysis, although the statement is true; choice-letter “c” is incorrect because total variable costs change (not unchanged) within the relevant range 8. At the breakeven point, the contribution margin equals total A. Variable costs. C. Selling and administrative costs.

Chapter 4 B.

Marginal costing and CVP analysis Sales revenues. (aicpa)

93

D. Fixed costs.

8. D ? The amount of contribution margin at breakeven point.  At breakeven point, there is no profit or loss, meaning, total sales equal total costs. Alternatively, profit is contribution margin less fixed costs and expenses. Therefore, at breakeven point contribution margin equals total fixed costs. Choice-letter “d” is correct. 9. At breakeven point, fixed cost is always A. Less than contribution margin. C. More than variable cost. B. Equal to contribution margin. D. More than the contribution margin.

(rpcpa)

9. B ? Fixed cost at breakeven point.  At breakeven point, total fixed cost equals contribution margin. This is so because, if fixed cost and contribution are of the same amount, there is no profit or loss. 10. An assembly plant accumulates its variable and fixed manufacturing overhead costs in a single cost pool, which is then applied to work-in-process using a single application base. The assembly plant management wants to estimate the magnitude of the total manufacturing overhead costs for different volume levels of the application activity base using a flexible budget formula. If there is an increase in the application activity base that is within the relevant range of activity for the assembly plant, which one of the following relationship regarding variable and fixed costs is correct? A. The variable cost per unit is constant, and the total fixed costs decrease. B. The variable cost per unit is constant, and the total fixed costs increase. C. The variable cost per unit and the total fixed costs remain constant. D. The variable cost per unit increases, and the total fixed costs remain constant. (cia) 10. C ? Relationship regarding variable and fixed costs within the relevant range.  Within the relevant range, unit variable cost and total fixed costs are constant. Choice-letter “c” is correct. 11. Contribution margin is the excess of revenues over A. Direct cost. C. Cost of good sold. B. Manufacturing cost. D. All variable costs.

(rpcpa)

11. D ? A procedure in computing contribution margin.  The traditional way to compute contribution margin is by getting the difference between net sales and variable costs and expenses. Other procedures to determine contribution margin (CM) are as follows: CM = Quantity sold x Unit contribution margin

Chapter 4

Marginal costing and CVP analysis

94

CM = Net sales x Contribution margin ratio CM = Fixed costs + Income before income tax Choice-letter “a” is incorrect because the difference between net sales and direct cost is direct margin or segment margin. Choice-letter “b” is also incorrect because the difference between revenues and manufacturing cost is manufacturing margin or manufacturing income. Choice-letter “c” is also incorrect because the difference between net sales and cost of goods sold is gross profit. 12. Cost-volume-profit analysis is a key factor in many decisions, including choice of product lines, pricing of products, marketing strategy, and use of productive facilities. A calculation used in a CVP analysis is the breakeven point. Once the breakeven point has been reached, operating income will increase by the A. Gross margin per unit for each additional unit sold. B. Contribution margin per unit for each additional unit sold. C. Variable cost per unit for each additional unit sold. D. Sales price unit for each additional unit sold. (cma) 12. B ? Effect to operating income, once the breakeven point has been reached.  Beyond the breakeven point, total sales are greater than total costs. Also, contribution margin is greater than fixed cost. The excess of contribution over fixed costs is profit. After breakeven point, income increases by every peso increase in contribution margin, which is the unit contribution margin. Choice-letter “b” is the right choice. 13.

Cost-volume-profit relationships that are curvilinear may be analyzed linearly by considering only A. Fixed and semi-variable costs. C. Relevant variable costs. B. Relevant fixed costs. D. A relevant range of volume. (aicpa)

13. D ? The factor to be considered in analyzing curvilinear relationships.  Choice-letter “d” is correct. Curvilinear relationships exist over the long run. In the short-term, however, the relationship between two variables in a graph is linear. This short-term range is otherwise known as relevant range and the linearity assumption is valid within the relevant range. It is a range of production activity where the behavior of costs and sales is linear; meaning, unit sales, unit variable cost, and total fixed cost are constant. Choice-letter “a” is incorrect because semi-variable costs are normally curvilinear in its representation. Choice-letters “b” and “c” are incorrect because other relevant costs are also included in the CVP analysis. 14. When an organization is operating above the breakeven point, the degree or amount that revenues may decline before losses are incurred is the A. Residual income rate. C. Margin of safety.

Chapter 4 B.

Marginal costing and CVP analysis Marginal rate of return. (cma)

D.

95

Target (hurdle) rate of return.

14. C ? The amount of decline in sales before losses occur.  If operations are in excess of breakeven point, there is profit. The amount of decline in sales before losses occur is margin of safety. Mathematically, margin of safety equals actual (or budgeted) sales less breakeven sales. 15.The rate or amount that sales may decline before losses are incurred is called: A. Sensitive level of income. C. Margin safety . B. Variable sales ratio. D. Residual income rates. (rpcpa) 15. C ? The rate or amount in sales before losses are incurred.  Margin of safety is the excess of sales over the breakeven point. Therefore, it is the maximum amount of reduction in sales before losses are incurred. Choice-letter “c” is correct. 16. Total unit costs are A. Relevant for cost-volume-profit analysis. B. Independent of the cost system used to generate them. C. Irrelevant in marginal analysis. D. Needed for determining product contribution. (rpcpa) 16. C ? A choice that describes total unit costs.  Choice-letter “c” is correct, total unit costs are irrelevant in marginal analysis. To be relevant in marginal costing analysis, total costs should be separated as to either fixed or variable. Total unit cost per se is not relevant in marginal costing analysis. Choice-letter “a” is incorrect. Total unit costs are dependent of the cost system used to generate them. Choice-letter “b” is incorrect. And total unit cost is not needed in determining the contribution margin, what is important is the variable cost. Choiceletter “d” is incorrect. Breakeven point 17. Given the following notations, what is the breakeven sales level in units? SP = selling price per unit FC = total fixed costs VC = variable cost per unit A. B.

SP ÷ (FC ÷ VC). FC ÷ [1 – (VC ÷ SP)]. (aicpa)

17. B ? Computation of breakeven point in units.

D.

C. VC÷ (SP – FC). FC ÷ (SP – VC).

Chapter 4

Marginal costing and CVP analysis

96

 Choice-letter “b” is correct. Unit contribution margin equals unit sales price (USP) less unit variable costs (UVC). Therefore, breakeven point (BEP) in units and in pesos are computed as follows: BEP (units) = Fixed costs / Unit contribution margin or FC / (USP – UVC) BEP (pesos) = Fixed costs / Contribution margin ratio or FC ÷ (100% - VCR) In the marginal costing analysis, sales ratio is always equal to 100%. Variable cost ratio (VCR) is basically variable cost over sales. 18. A company’s breakeven point (BEP) in pesos of revenue may be affected by equal percentage increase in both selling price and variable cost per unit (assume all other factors are constant within the relevant range). The equal percentage changes in selling price and variable cost per unit will cause the breakeven point in pesos to A. Decrease by less than the percentage increase in selling price. B. Decrease by more than the percentage increase in the selling price. C. Increase by the percentage change in variable cost per unit. D. Remain unchanged. (cia) 18. D ? Effect to breakeven point of equal percentage increase in both sales price and unit variable cost.  The equal percentage increase in both sales price and unit variable cost does not change the contribution margin ratio. Therefore, the breakeven point will not change, assuming all other factors are constant. Choice-letter “d” is correct. Questions 19 and 20 are based on the following selected budgeted data of Russel Gil Company for the coming year: Selling price per unit P 12.00 Budgeted sales 600,000 Fixed expenses 150,000 Variable cost per unit 8.00 19. What is the breakeven in sales in units? A. 35,000 B. 37,500

C. 40,000 D. 45,000

19. B ? Breakeven sales in units.  Breakeven sales in units equals total fixed costs over unit contribution margin. The unit contribution margin is P4.00 (i.e., P12 – P8). Therefore, breakeven sales in units is 37,500, computed as follows: BEP (units) = P150,000 / P4 = 37,500 units 20. What is the margin of safety ratio in percent? A. 15% C. 30%

Chapter 4 B.

Marginal costing and CVP analysis 20% (rpcpa)

97

D, 25%

20. D ? Margin of safety ratio.  Margin of safety ratio (MSR) is margin of safety divided by actual (or budgeted) sales. Considering the given data, we have: Budgeted sales P 600,000 Less: Breakeven sales (37,500 units x P12) 450,000 Margin of safety P 150,000 Therefore, MSR is 25% (i.e., P150,000 / P600,000). 21. A company is concerned about its operating performance, as summarized below: Revenues (P12.50 per unit) P300,000 Variable costs 180,000 Operating loss (40,000) How many additional units should have been sold in order for the company to break even in 2004? A. 32,000 C. 16.000 B. 24,000 D. 8.000 (cia) 21. D ? The number of additional units to sell to breakeven.  The company is operating at a loss. It has to increase its actual units sold to breakeven. The additional units to sell to breakeven is the difference between breakeven point and the present actual units sold. The variable cost ratio is 60% (e.g., P180,000/P300,000). Therefore, the CMR is automatically 40%, and the UCM is P5 (e.g., P12.50 x 40%). Total fixed cost equals CM plus operating loss. CM is P120,000 (e.g., P300,000 – P180,000). Therefore, total fixed cost is P160,000 (e.g., P120,000 + P40,000). The additional units to sell is calculated as follows: Breakeven sales (P160,000 / P5) 32,000 units - Actual units sold (P300,000 / P12.50) 24,000 Needed increase in units sold to breakeven 8,000 units Alternatively, the increase in units needed to breakeven is 8000 units determined by dividing the loss by the unit contribution margin (i.e., P40,000 / P5 = 8,000 units) 22. Kent Co.'s operating percentages were as follows: Revenues Cost of good sold Variable 50% Fixed 10 Gross profit Other operating expenses Variable 20 Fixed 15

100% 60 40% 35

Chapter 4

Marginal costing and CVP analysis

98

Operating income 5% Kent’s sales totaled P2 million. At what revenue level would Kent break even? A. P1,900.000 C. P1,250.000 B. P1,666.667 D. P 883,333 (aicpa) 22. B ? Breakeven sales.  BEP is fixed costs divided by CMR. Total fixed cost is P500,000 (i.e., P2,000,000 x 25%). The total variable cost ratio is 70% (i.e., 50% + 20%). Therefore, CMR is 30%. The breakeven point in pesos is: BEP (pesos) = P500,000 / 30% = P1,666,667 23. For the period just ended Chanda, Inc., generated the following operating results in percentages Sales 100% Cost of sales Variable 50% Fixed 10% 60% Gross profit 40% Operating expense Variable 20% Fixed 15% 35% Operating income 5% Total sales amounted to P3.0 million. at what level is break-even sales? A. P3,750,000 C. P1,875,000 B. P1,850,000 D. P2,500,000

(rpcpa)

23. B ? The breakeven sales in peso.  BEP in pesos equals fixed costs divided by CMR. The fixed cost rate on sales totals 25% (fixed overhead of 10% plus fixed expenses of 15%). Therefore, total fixed costs and expenses equal P750,000 (P3 million x 25%). The CMR is 30% (that is, 100% sales ratio less 70% total variable costs and expenses ratio). Finally, BEP in pesos is P2,500,000 (P750,000 / 30%). 24. A company produced 500 units of a product and incurred the following costs. Direct materials, P8,000; direct labor, P10,000 overhead (20% fixed), P45,000. If the sales value of 500 units is P102,000, what is the contribution margin percentage? A. 44% C. 53% B. 47% D. 74% (rpcpa) 24. B ? The contribution margin percentage.

Chapter 4

Marginal costing and CVP analysis

99

 Contribution margin percentage (CMR) is the quotient of contribution margin and net sales. Given the data on the problem, the contribution margin is determined as follows: Sales P102,000 Direct materials ( 8,000) Direct labor ( 10,000) Variable overhead (P45,000 x 80%) ( 36,000) Contribution margin P 48,000 The CMR is 47% (i.e., P48,000/P102,000). 25. Given the selling price at P120 per unit; contribution margin ratio at 25% and fixed cost at P250,000, the total variable expenses at the break even point would be: A. P350,000 C. P450,000 B. P750,000 D. P250,000 (rpcpa) 25. B ? The amount of variable expenses at breakeven point.  Based on the data given, the variable expenses at the BEP may be computed by multiplying breakeven sales and VCRatio. BEP is determined as fixed cost divided by CMRatio. BEP (P250,000/25%) P1,000,000 x VCRatio (100%- 25%) 75% Variable expenses P 750,000 Or variable expenses (P250,000 / 25% x 75%)

P 750,000

26. Which of the following formulas is used to determine the break-even point when using the contribution margin method? A. Revenues less operating income equals variable costs plus fixed costs. B. Unit contribution margin times the break-even number of units equals fixed costs. C. Selling price less unit fixed costs equals contribution margin D. Total fixed costs equal total revenues. (rpcpa) 26. B ? The formula used in determining the breakeven point (BEP).  Choice-letter “b” is correct. At BEP, total fixed costs equal to contribution margin where fixed costs equal unit contribution margin times the number of breakeven units. Choice-letter “a” is incorrect because it does not have relevance in the planning and control of costs and profit. Choice-letter “c” is incorrect because sales less fixed costs is the sum of variable costs and profit (it also has no relevance). Choice-letter “d” is definitely incorrect. 27. Which of the following will result in raising the breakeven point? A. A decrease in the variable cost per unit. B. An increase in the semi-variable cost per unit. C. An increase in the contribution margin per unit.

Chapter 4

Marginal costing and CVP analysis

D. A decrease in income tax rates.

100 (cia)

27. B ? The one that will raise breakeven point.  BEP is fixed costs over UCM. To increase BEP, fixed cost should increase and unit contribution margin should decrease. Choice-letter “b” is correct, an increase in semi-variable cost would decrease unit contribution margin, and increase in BEP. A semi-variable cost per unit is more of a variable cost than that of fixed costs. Choice-letters “a” and “c” increase unit contribution margin resulting to decreased BEP. Choice letter “d” will not affect BEP. 28. To reduce the break-even point, the company may A. Decrease both the fixed costs and contribution margin. B. Increase both the fixed costs and the contribution margin. C. Decrease the fixed costs and increase the contribution margin. D. Increase the fixed costs and decrease the contribution margin.

(rpcpa)

28. C ? The technique in reducing the breakeven point (BEP).  Breakeven point is fixed costs over unit contribution margin (UCM). Therefore, to reduce the BEP, fixed costs must be decreased or UCM must be increased (choiceletter “c”: is correct). 29. NTQ Inc’s net sales in 2006 were 15% below the 2005 level. NTQ’s semi-variable costs would A. Increase in total and increase as a percentage of net sales. B. Decrease in total and decrease as a percentage of net sales. C. Increase in total, but decrease as a percentage of net sales. D. Decrease in total, but increase as a percentage of net sales. (rpcpa) 29. D ? Effect to semi-variable cost if net sales decrease by 15%.  Total semi-variable costs change, but not in direct relation to the change amount of sales (say, if net sales increase by 10%, semi-variable costs may increase but not by 10%). Therefore, if net sales decrease by 15%, the total semi-variable costs will also decrease but its percentage relationship to net sales will increase because the percentage decrease in net sales is greater than the percentage decrease in semivariable costs. 30. Cost-volume-profit analysis is a key factor in many decisions, including choice of product-lines, pricing of products, marketing strategy, and utilization of productive facilities. A calculation used in CVP analysis is the break-even point. Once the breakeven point has been reached operating income will increase by the A. Sales price per unit for each additional unit sold. B. Contribution margin per unit for each additional unit sold. C. Fixed cost per unit for each additional unit sold. D. Gross margin per unit for each additional unit sold. (rpcpa)

Chapter 4

Marginal costing and CVP analysis

101

30. B ? The increase in operating income once the breakeven point (BEP) is reached.  Choice-letter “b” is correct; once the BEP is reached, the operating income will increase by every peso of increase in the amount of contribution margin. Total contribution margin is equal to unit contribution margin (UCM) multiplied by the units sold. At BEP, total fixed cost is equal to total contribution margin. Fixed costs are assumed to be constant even beyond the BEP. Therefore, after the BEP, as contribution margin increases (due to the continuing increase in the number of units sold) with fixed costs as constant, any increase in the contribution margin is an increase in profit. Choice-letter “a” is incorrect because sales price is only an increase in revenue and not yet in profit. Choice-letter “c” is incorrect because fixed cost per unit does not affect the overall profit of the business, though it has great impact in the establishment of a competitive unit sales price. Choice-letter “d” is incorrect because unit gross margin does not reflect yet the net profit as variable selling and administrative expenses are still to be deducted. 31. Marston Enterprises sells three chemicals: petrol, septine and tridol. Petrol is the company’s most profitable product; tridol is the least profitable. Which one of the following events will definitely decrease the firm’s overall breakeven point for the upcoming accounting period? A. The installation of new computer-controlled machinery and subsequent layoff of assembly-line workers. B. A decrease in tridol’s selling price. C. An increase in anticipated sales of petrol relative to sales of septine and tridol. D. An increase in petrol’s raw material cost. (cma) 31. C ? The event that will decrease BEP.  Choice-letter “c” is correct because an increase in the anticipated sales of petrol relative to sales of septine and tridol will increase the weighted average unit contribution margin, which will decrease BEP. Choice-letter “a” would increase fixed cost and breakeven point. Choice-letters “b” and “d” reduce unit contribution margin and increase BEP. 32. Cook Company sells three chemicals; Simpol, Plutex and Coplex. Simpol is the most profitable product while Coplex is the least compatible. Which of the following events will definitely decrease the firm’s overall BEP for the upcoming account period. A. An increase in the overall market of Plutex B. An decrease in Coplex’s selling price C. An increase in anticipated sales of Simpol relative to the sales of Plutex and Coplex D. An increase in Simpol raw materials (rpcpa) 32. C ? The event that will decrease the overall breakeven point (BEP).

Chapter 4

Marginal costing and CVP analysis

102

 Composite (or overall) BEP is equal to fixed costs divided by the average UCM. To decrease the composite BEP, the fixed costs must decrease or the average UCM must increase. The company produces and sells three products. Simpol is the most profitable, Coplex is the least profitable, and Phines is at the middle. Choice-letter “c” is correct because an increase in the anticipated sales of Simpol relative to the sales of Plutex and Coplex would trigger an increase in the average UCM, and thereby would decrease composite BEP. Choice-letter “a” is incorrect because an increase in overall market for Phines, the product at the middle of the sales mix profitability, would hardly change the average UCM and that of breakeven point. Choice-letter “b” is incorrect because a decrease in Coplex’s selling price would aggravate a decrease in the average UCM and will increase the composite BEP. Choice-letter “d” is also incorrect because an increase in Simpol’s raw materials cost would mean a decrease in UCM and an increase in the composite BEP. 33. A company manufactures a single product. Estimated cost data regarding this product and other information for the product and the companies are as follows: Sales price per unit P40 Total variable production cost per unit P22 Sales commission (on sales) 5% Fixed costs and expenses: Manufacturing overhead P5,598,720 General and administrative P3,732,480 Effective income tax rate 40% The number of units the company must sell in the coming year in order to reach its breakeven point is A. 388,800 units. C. 583,200 units. B. 518,400 units. D. 972,000 units. (cia) 33. C ? The number of units to breakeven.  The sales commission is P2 per unit (5% x P40). The total unit variable cost is P24 (i.e., P22 + P2), and the unit contribution margin is P16 (i.e., P40 – P24). Total fixed costs is P9,331,200 (i.e., P5,598,720 + P3,732,480). Therefore, the BEP in units is: BEP (units) = P9,331,200 / P16 = 583,200 units 34. Fely Company reported the following for the year just ended: Budgeted sales P 3,000,000 Break-even sales 2,100,000 Budgeted contribution margin 1,800,000 Cashflow break-even 600,000 The company’s margin of safety is A. P 900,000 B. P2,400,000 (rpcpa)

C. P1,200,000 D. P1,500,000

Chapter 4

Marginal costing and CVP analysis

103

34. A ? The amount of margin of safety.  Margin of safety is the difference between actual of budgeted sales and breakeven sales, as follows: Budgeted sales P3,000,000 - Breakeven sales 2,100,000 Margin of safety P 900,000 35. The contribution margin ratio always increases when the A. Breakeven point increases. B. Breakeven point decreases. C. Variable costs as a percentage of net sales decrease. D. Variable costs as a percentage of net sales increase. (aicpa) 35. C ? The event that makes the contribution margin ratio (CMR) to increase.  CMR increases when unit sales price increases or unit variable cost decreases, or, in effect, when unit contribution margin increases. Choice-letter “c” is correct, as variable cost ratio decreases, automatically CMR increases. CMR affects breakeven point and not the BEP affecting the CMR. Besides, the change in BEP is not automatically due to the change in the CMR. Choice-letters “a” and “b” are incorrect. 36. The following information pertains relationships: Breakeven point in units sold Variable costs per unit Total fixed costs

to

Nova

Company’s

cost-volume-profit

1,000 P 500 P150,000

How much will be contributed to profit before income taxes by the 1,001st unit sold? A. P650 C. P150 B. P500 D. P 0 (aicpa) 36. C ? The contribution to profit by the 1,001st unit sold.  The 1,001st unit sold is the very first unit after the breakeven point. Beyond the breakeven point, profit increases by every peso of increase in contribution margin. The profit contributed by the first unit after the BEP is the unit contribution margin. At breakeven point, contribution margin equals fixed costs. And unit contribution margin is total fixed costs divided by breakeven units. The unit contribution margin is determined as follows: UCM = Fixed costs / BEP units UCM = P150,000 / 1,000 units = P150 37. The following data refers to cost volume profit relationships of Trilogy Co:

Chapter 4

Marginal costing and CVP analysis

Breakeven point in units 1,000 Variable costs per unit P 250 Total fixed costs P75,000 How much will be contributed to operating income by the 1,001st unit sold? A. P250 C. P75 B. P325 D. Zero

104

(rpcpa)

37. C ? The contribution to operating income by the 1,001st unit sold.  The 1,001st unit sold is the first unit sold after the breakeven point. After the breakeven point, each unit sold increases profit by the amount of the unit contribution margin (UCM). The UCM is equal to contribution margin over the number units sold. At the BEP, the CM is equal to fixed cost. The fixed cost is given at P75,000. The UCM is P75 and this is also the amount of profit on the 1,001st unit sold. UCM = P75,000 / 1,000 units = P75 38. A retail company determines its selling price by marking up variable costs by 60%. In addition, the company uses frequent selling price markdown to stimulate sales. If the markdown average 10%, what is the company contribution margin ratio? A. 27.5% C. 37.5% B. 30.6% D. 41.7% (cia) 38. B ? The contribution margin ratio.  The sales price is based on variable cost, which is the 100%. Variable cost ratio is variable cost over net sales price. Net sales price is 160% of variable cost less 10% markdown. The variable cost ratio (VCR) and contribution margin ratio (CMR) are calculated as follows: VCR = 100% / (160% x 90%) = 69.4%. And, CMR = 100% - 69.44% = 30.6% 39. Which of the following would decrease unit contribution margin the most? A. A 15% decrease in selling price. C. A 15% decrease in variable costs. B. A 15% increase in variable costs. D. A 15% decrease in fixed costs. (cma) 39. A ? The one that decreases contribution margin the most.   Contribution margin is the difference between sales and variable cost. Contribution margin is decreased by a decrease in sales price, an increase in variable cost or a decrease in unit sales price and unit variable cost by the same amount. Choice-letters “a” and “b” would decrease contribution margin. Inasmuch as the sales price is assumed to be greater than the variable cost, a 15% decrease in sales price (choice-letter “a”) would decrease contribution margin more than the decrease effected by a 15% increase in variable cost (choice-letter “b”). Choice-letter “a” is the correct choice.

Chapter 4

Marginal costing and CVP analysis

105

Choice-letter “c” would increase contribution margin, while choice-letter “d” does not affect contribution margin. Questions 40 and 41 are based on the following information. Lan Pala Tropical Stuff Toys manufactures and sells dolls. The following information relates to the operating results for the last quarter: Stuff toys sold 19,375 Breakeven point in number of toys 15,500 Breakeven point in peso sales P65,875 Total fixed costs P47,275 40. What was Lan’s variable cost per doll? A. P 4.25 B. P 3.05 (rpcpa)

C. P 1.20 D. P 0.96

40. C ? The amount of variable cost per doll.  Unit variable cost is the difference between unit sales price and unit contribution margin. At breakeven point, fixed costs equal the contribution margin. Therefore, the unit variable cost is computed as follows: Unit sales price (P65,875 / 15,500) P4.25 - Unit contribution margin (P47,275 / 15,500) 3.05 Unit variable cost P1.20 41. What was the margin of safety percentage for the last quarter of Lan? (rounded to the nearest percent) A. 20% C. 28% B. 25% D. 72% (rpcpa) 41. A ? The margin of safety ratio (MSR).  Margin of safety ratio is margin of safety over sales. First, let us get the margin of safety, then the margin of safety ratio. Actual sales (19,375 x P4.25) - Breakeven sales Margin of safety

Amount P82,343.75 65,875.00 P16,468.75

Units 19,375 15,500 3,875

The margin of safety ratio is 20% (i.e., P16,468.75 / P82,343.75). Alternately, the MSR may be determined using the data in units and still arrive at 20% (i.e., 3,875/19,375). 42. For a profitable company, the amount by which sale can decline before losses occur is known as the A. Variable sales ratio. C. Sales volume variance.

Chapter 4 B.

Marginal costing and CVP analysis Margin of safety. (rpcpa)

D.

106

Marginal income rate.

42. B ? The amount of decrease in sales before losses occur.  Losses occur when sales fall below the breakeven point (BEP). At BEP, there is still no loss. Therefore, the amount of decline in sales before losses occur is the difference between the actual (or budgeted) sales and the breakeven sales. This is called the margin of safety (choice-letter “b” is correct). Analyzing it in a different perspective, margin of safety is the amount of sales in excess of BEP. It suggests that this excess sales carries a corresponding contribution margin, and eventually, profit (e.g., Profit = Margin of Safety x CMR). Choice-letter “a” is incorrect because variable sales ratio (or variable ratio) is the relationship of variable costs over sales. Choice-letter “c” is incorrect because sales volume variance refers to the net effect on profit of the difference in actual sales and budgeted sales. Choice-letter “d” is incorrect because marginal income rate is a loose description that relates to the relationship of margin over sales and not on the amount of a reduction in sales. 43. Bulacan Gold, Inc. manufactures and sells key rings embossed with college names and slogans. Last year, the key rings sold for P75 each, and the variable costs to manufacture them were P22.50 per unit. The company needed to sell 20,000 key rings to break-even. The net income last year was P50,400. The company expects the following for the coming year: • The selling price of the key rings will be P90. • Variable manufacturing costs per unit will increase by one-third. • Fixed cost will increase by 10%. • The income tax rate will remain unchanged. For the company to break-even the coming year, the company should sell A. 21,600 C. 21,250 B. 2,600 D. 19,250 (rpcpa) 43. D ? The breakeven point in the coming year.  Breakeven point is fixed costs divided by unit contribution margin. Using the “beforeafter analysis”, the data are treated as follows: Before After Unit sales price P 75.00 P 90.00 Unit variable costs 22.50 30.00 (P22.50 x 1.33) Unit contribution margin P 52.50 P 60.00 Total fixed costs (20,000 x P52.50) P1,050,000 P1,155,000 (P1,050,000 x 110%) The new breakeven point in the coming year shall be 19,250 units (P1,155,000 / P60). 44. A company has revenues of P500, 000, variable costs of P300, 000, and pretax profit of P150, 000. If the company increased the sales price per unit by 10%, reduced

Chapter 4

Marginal costing and CVP analysis

107

fixed costs by 20%, and left variable cost per unit unchanged, what would be the new breakeven point in pesos? A. P 88,000 C. P110,000 B. P100,000 D. P125,000 (cia) 44. A ? The new breakeven point in pesos.  Inasmuch as there are changes in the variables of profit, the before-after analysis would be used as follows: Before After Sales P500,000 P550,000 (P500,000 x 110%) -Variable cost 300,000 300,000 (same) Contribution margin P200,000 P250,000 CMR 45.45% (P250,000 / P550,000) Fixed costs P 50,000 P 40,000 (P50,000 x 80%) New BEP (pesos) P 88,000 (P40,000 / 45.45%) 45. Mela Corporation has a contribution margin ratio of 0.26. It aims to have a net income of P320,000 with a sales volume of P2 million. Its total fixed costs amount to A. P200,000 C. P230,777 B. P 83,200 D. P520,000 (rpcpa) 45. A ? The amount of fixed costs.  Fixed costs and expenses may be determined by getting the difference of contribution margin and income before income tax, as follows: Contribution margin (P2 million x .26) P 520,000 - Income before income tax 320,000 Total fixed costs and expenses P 200,000 46. Asian Corporation, a manufacturing company, is operating at 90% capacity. Since there is no other use of the 10% idle capacity, an offer for a new order at P8.20 per unit requiring 15% capacity is being considered. If the order will be accepted, the 5% additional capacity will be sub-contracted at the cost of P7.80 per unit. The variable cost per unit of production of Asian Corporation follows: Materials P 4.00 Labor 1.75 Variable overhead 1.75 Total P 7.50 What is the expected contribution margin per unit on the new order? A. P 0.40 C. P 0.50 B. P 0.60 D. P 0.55 (rpcpa) 46. B

Chapter 4

Marginal costing and CVP analysis

108

? The expected unit contribution margin on the new order.  There are two unit contribution margin here. One for the 10% capacity and the other for the 5% capacity, as follows: 10% capacity 5% capacity (Regular) (Sub-contract) Unit contribution margin P8.20 P8.20 - Unit variable costs 7.50 7.80 Unit contribution margin P0.70 P0.40 The individual UCM shall be multiplied by the production mix of 10% and 5% for regular production and sub-contract, respectively. The average unit contribution margin may now be determined as: Regular sales (P0.70 x 10/15) P0.47 Sub-contract (P0.40 x 5/15) 0.13 Average unit contribution margin P0.60 47. Last year, the contribution margin ratio of Lamesa Company was 30%. This year, fixed costs are expected to be P120, 000, the same as last year, and revenues are forecasted at P550, 000, a 10% increase over last year. For the company to increase operating income by P15,000 in the coming year, the contribution margin ratio must be A. 20% C. 40% B. 30% D. 70% (aicpa) 47. B ? The contribution margin ratio to increase operating income by P15,000.  The new contribution margin ratio is determined by dividing the new contribution margin with the new sales. The new contribution margin is calculated as follows: Sales last year (P550,000/110%) P500,000 x CMR last year 30% CM last year 150,000 Add: Increase in contribution margin 15,000 CM this year 165,000 Divide by sales this year 550,000 CMR this year 30% 48. A company increased the selling price of its product from P1.00 to P1.10 a unit when total fixed costs increased from P400,000 to P480,000 and variable cost per unit remained unchanged. How will these changes affect the breakeven point? A. The breakeven point in units will be increased. B. The breakeven point in units will be decreased. C. The breakeven point in units will remain unchanged. D. The effect cannot be determined from the information given. (aicpa) 48. D ? Effects to BEP of a change in unit sales price (USP) and fixed costs (FC).

Chapter 4

Marginal costing and CVP analysis

109

 An increase in USP increases unit contribution margin (UCM) and decreases BEP, while an increase in FC increases BEP. The unit variable cost remains constant and there is no exact variable cost data given to precisely determine the quantitative effects of the given change in USP and FC to BEP. The effect cannot be determined from the information given because the amount of unit variable cost also affects the change in the BEP. To illustrate, let us use the following data: a.) Say, a unit variable cost of P 0.70: Original Data New Data Unit sales price P1.00 P1.10 Unit variable cost 0.70 0.70 Unit contribution margin P0.30 P0.40 Fixed costs P 400,000 P 480,000 Breakeven units 1,333,333 1,200,000 (BEP decreases) b.) Say, a unit variable cost of P0.40: Original Data Unit sales price P1.00 Unit variable cost 0.40 Unit contribution margin P0.60 Fixed costs P 400,000 Breakeven units 666,667

New Data P1.10 0.40 P0.70 P 480,000 685,715 (BEP increases)

49. Two companies produce and sell the same product in a competitive industry. Thus, the selling price of the product of each company is the same. Company 1 has a contribution margin ratio of 40% and fixed costs of P25 million. Company 2 is more automated, making its fixed costs 40% higher than those of Company 1. Company 2 also has a contribution margin ratio that is 30% greater than that of Company 1. By comparison, Company 1 will have the breakeven point in terms of pesos sales volume and will have the peso profit potential once the indifference point in peso sales volume is exceeded. List A List B List A List B A. Lower Lesser C. Higher Lesser B. Lower Greater D. Higher Greater (cia) 49. A ? Comparing Company 1 to Company 2, their BEP and effects to operating profit beyond the BEP.  Company 1 has a higher CMR and lower fixed costs which yield to lower BEP compared that of Company After the BEP, the company having higher CMR is expected to register higher profit (i.e., Profit = Margin of safety x CMR). Since Company 2 has a higher profit (e.g., 40%) than that of Company 1 (e.g., 30%), its increase in profit will tend to be greater after the BEP. Sales with profit 50. In using cost-volume-profit analysis to calculate expected unit sales, which of the following should be deducted to fixed cost in the numerator? A. Predicted operating loss. C. Unit contribution margin.

Chapter 4 B.

Marginal costing and CVP analysis Predicted operating income. (aicpa)

110

D. Variable costs.

50. A ? The one added to fixed costs in the numerator in CVP analysis.  Choice-letter “a is correct, the predicted operating loss is deducted from the fixed costs in the numerator in computing sales with profit. Choice-letter “b” is incorrect because operating income is added from fixed costs in the numerator in calculating sales with profit. Choice-letter “c” is incorrect because unit contribution margin serves as the denominator in the computation of sales in units. Choice-letter “d” is incorrect because variable cost is not related to fixed cost. 51. Ipil-ipil Company would like to market a new product at a selling price of P15 per unit. Fixed costs for his product are P1,000,000 for less than 500,000 units of output and P1,500,000 for 500,000 or more units of output. The contribution margin percentage is 20%. How many units of this product must be sold to earn a target operating income of P1 million? A. 754,900 C. 825,530 B. 833,334 D. 785,320 (rpcpa) 51. B ? The number of units to be sold to earn an income of P1 million.  There are two fixed costs given, P1 million at less than 500,000 units of production and P1.5 million at 500,000 units or more. The UCM is P3.00 (P15 x 20%). If the company produces and sells 500,000 units, the operating income shall be P500,000. That is, CM of P1,500,000 (500,000 units x P3) less fixed costs of P1,000.000. Therefore, to generate an earnings of P1,000,000, the company must product and sell more than 500,000 units and the fixed cost to be incurred shall be P1,500,000. The sales in units to product a profit of P1,000,000 is 833,334 units computed as follows: Sales = FC + IBIT / UCM = (P1,500,000 + P1,000,000) / P3 = 833,334 units 52. Merchandiser, Inc. sells Product O to retailers for P200. The unit variable cost is P40 with a selling commission of 10%. Fixed manufacturing costs total P1,000,000 per month while fixed selling and administrative costs total P420,000. The income tax rate is 30%. The target sales if after tax income is P123,200 would be A. 10,950 units. C. 13,750 units. B. 15,640 units. D.11,400 units. (rpcpa) 52. D ? The target sales if the after tax income is P123,200.  The total unit variable cost is 60 [i.e., P40 + (10% x P200)]. And the UCM is P140 (i.e., P200 – 60). The total of fixed costs and expenses is P1,420,000 (i.e., P1,000,000 + P420,000). And IBIT is P176,000 (i.e., P123,200 / 70%). Therefore, the targeted sales in units is 11,400 units computed as follows: Sales = (FC + IBIT) / UCM

Chapter 4

Marginal costing and CVP analysis

111

Sales = (P1,420,000 + P176,000) / P140 = 11,400 units 53. Nette & Company has sales of P400,00 with variable costs of P300,000, fixed costs of P120,000 and an operating loss of P20,000. By how much would Nette need to increase its sales in order to achieve a target operating income of 10% of sales? A. P400,000 C. P500,000 B. P462,000 D. P800,000 (rpcpa) 53. A ? The amount of increase in sales to achieve a target profit of 10% of sales.  The present sales amount to P400,000, the VCR is 75% (i.e., P300,000/P400,000), and, therefore, the CMR is 25%. Since the profit ratio is given, the computation of the sales with profit shall be: Sales = Fixed costs / (CMR – NPR) = P120,000 / (25% - 10%) = P800,000 The increase in sales shall therefore be P400,000 (i.e., P800,000 – P400,000). Questions 54 and 55 are based on the following information. Laguna Marketing Company is expected an increase of fixed costs by P78,750 upon moving their place of business to the downtown area. Likewise, it is anticipating that the selling price per unit and the variable expenses will not change. At the present, the sales volume necessary to breakeven is P750,000 but with the expected increase in final sales, the sales volume necessary to breakeven would go up to P975,000. Based on these projections,

54. What is the profit-volume ratio of Laguna Marketing? A. 35% C. 45% B. 40% D. 65% (rpcpa) 54. A ? The profit-volume ratio.  The profit-volume ratio is the same as the contribution margin ratio. And CMR is contribution margin divided by sales. Note that sales given in the question are at breakeven point. This means that at these sales there is no profit or loss and the change in profit is also zero. Therefore, any change in fixed cost is also the change in contribution margin. The CMR of 35% is determined as follows: CMR = Δ FC / Δ Sales (i.e., the sales are at BEP) = P78,750 / (P975,000-P750,000) = 35% 55. What would be the total fixed costs of Laguna Marketing after the increase of P78,750? A. P341,250 C. P2,183,750 B. P262,500 D. P 300,000 (rpcpa) 55. A ? The fixed cost after the increase of P78,750.

Chapter 4

Marginal costing and CVP analysis

112

 The given sales are at breakeven point. At BEP, fixed cost equals contribution margin. The fixed costs after its increase shall be the contribution margin in the second year with breakeven sales of P975,000. The fixed costs P341,250 (i.e., P975,000 x 35%). 56. Variable cost per unit is P3.50. Contribution margin is 30%. Breakeven sales is P1 million. To sell an additional 50,000 units at the same price and contribution margin, how much will fixed costs increase to have a gross margin equal to 10% of the sales value of the additional cost of 50,000 units to be sold? A. P 67,500 C. P 57,500 B. P 50,000 D. P 125,000 (rpcpa) 56. B ? The amount of increase in fixed cost.  Increase in contribution margin less increase in operating income is increase in fixed cost. The unit sales price is P5 (i.e., P3.50 / 70%) and the UCM is P1.50 (P5 – P3.50). The gross margin (e.g., operating income) is 10% of sales. The increase in fixed cost is P50,000, calculated as follows: Increase in CM (50,000 units x P1.50) P75,000 - Increase in operating income [50,000 units x (10% x P5)] 25,000 Increase in fixed cost P50,000 57. Birney Company is planning its advertising campaign for 2006 and has prepared the following budget data based on a zero advertising expenditures: Normal plant capacity 200,000 units Sales 150,000 units Selling price P25 per unit Variable manufacturing costs P15 per unit Fixed costs: Manufacturing P 800,000 Selling and administration P 700,000 An advertising agency claims that an aggressive advertising campaign would enable Birney to increase its unit sales by 20%. What is the maximum amount that Birney can pay for advertising and obtain an operating profit of P200,000. A. P100,000 C. P300,000 B. P200,000 D. P550,000 (aicpa) 57. A ? The maximum amount of advertising expense to obtain a profit of P200,000.  Increase in fixed cost is new fixed cost less old fixed cost. The old fixed cost is P1,500,000 (i.e., P800,000 + P700,000). The new fixed cost is not readily given and must be computed. The UCM is P10 (i.e., P15 – P10). And sales are expected to increase by 20%.

Chapter 4

Marginal costing and CVP analysis

The increase in fixed cost shall be computed as follows: New contribution margin (50,000 units x 120% x P15) - New operating income New fixed costs - Old fixed costs Increase in fixed costs

113

P1,800,000 200,000 1,600,000 1,500,000 P 100,000

Questions 58 through 60 are based on the following information. Bruell Electronics Co. is developing a new product, surge protectors for high-voltage electric flows. The cost information below relates to the product. Unit Costs Direct materials P 3.25 Direct labor 4.00 Distribution .75 The company will also be absorbing P120,000 of additional fixed costs associated with this new product. A corporate fixed charge of P20,000 currently absorbed by other products will be allocated to this new product. 58. If the selling price is P14 per unit, the breakeven point in units (rounded to the nearest hundred) for surge protectors is A. 8,600 units. C. 15,000 units. B. 10,000 units. D. 20,000 units. (cma) 58. D ? Breakeven point in units.   The unit variable cost is P8 (i.e., P3.25 + P4.00 + P0.75), and the unit contribution margin is P6 (i.e., P14 – P8). Therefore, the BEP in units is 20,000 units (i.e., P120,000/P8). Allocated fixed costs are not included in the determination of the product’s breakeven point. Allocated fixed costs, however, shall be included in the total fixed costs in the computation of overall company’s breakeven point. 59. How many surge protectors (rounded to the nearest hundred) must Bruell Electronics sell at a selling price of P14 per unit to gain P30,000 additional operating income before taxes? A. 10,700 units. C. 25,000 units. B. 20,000 units. D. 28,300 units. (cma) 59. C ? Total sales in units if operating profit is P30,000.  Sales in units are fixed costs plus operating profit divided by unit contribution margin. Therefore, sales is unit is 25,000 units, computed as follows: Sales = (FC + IBIT) / UCM Sales = (P120,000 + P30,000) / P6 = 25,000 units

Chapter 4

Marginal costing and CVP analysis

114

60. How many surge protectors (rounded to the nearest hundred) must Bruell Electronics sell at a selling price of P14 per unit to increase after-tax income by P30,000? Bruell Electronics’ effective income tax rate is 40%. A. 10,700 units. C. 25,000 units. B. 20,000 units. D. 28,300 units. (cma) 60. D ? Sales in units if the after-tax income is P30,000.  If the after-tax income is P30,000, the income before income tax is P50,000 (i.e., P30,000 / 60%). Therefore, the sales in units is 28,334 units, as follows: Sales in units = (P120,000 + P50,000) / P6 = 28,334 units 61. Austin Manufacturing, which is subject to a 40% income tax rate, had the following operating data for the period just ended: Selling price per unit P 60 Variable cost per unit 22 Fixed costs 504,000 Management plans to improve the quality of its sole product by (1) replacing a component that costs P3.50 with a higher-grade unit that costs P5.50 and (2) acquiring a P180, 000 packing machine. Austin will depreciate the machine over a 10-year life with no estimated salvage value by the straight-line method of depreciation. If the company wants to earn after tax income of P172, 800 in the upcoming period, it must sell A. 19,300 units. C. 22,500 units. B. 21,316 units. D. 27,000 units. (cma) 61. C ? Sales in units if the after-tax income is P172,800.   The income before income tax is P288,000 (i.e., P172,800 / 60%). There is an increase in unit variable cost of P2 (i.e., P5.50 – P3.50), and the new unit variable cost is P24 (i.e., P22 + P2). The sales price remains the same, therefore, the new unit contribution margin is P36 (P60 – P24). Total fixed cost is increased by P18,000 (i.e., P180,000/10 years). Hence, the new fixed cost is P522,000 (i.e., P504,000 + P18,000). The sales to achieve an aftertax income of P172,800 is 22,500 units, calculated as follows: Sales (units) = (P522,000 + P288,000) / P36 = 22,500 units Questions 62 and 63 are based on the following information. An organization sells a single product for P40 per unit that it purchased for P20. The salespeople receive a salary plus a commission of 5% of sales. Last year the organization’s net income (after taxes) was P100,800. The organization is subject to an income tax rate of 30%. The fixed costs of the organization are Advertising P 124,000 Rent 60,000 Salaries 180,000

Chapter 4 Other fixed costs Total

Marginal costing and CVP analysis

115

32,000 P396,000

62. The breakeven point in unit sales for the organization is A. 8,800 units. C. 19,800 units. B. 18,000 units. D. 22,000 units. (cia) 62. D ? Breakeven point in units.  The unit variable cost is P22 composed of unit purchase cost of P20 and commission of P2 (i.e., 5% x P40). The unit contribution margin is P18 (I.e., P40 – P22). And the BEP in units is 30,000 units (i.e., P396,000 ÷ P18). 63. The organization is considering changing the compensation plan for sales personnel. If the organization increases the commission to 10% of revenues and reduces salaries by P80,000, what revenues must the organization have to earn to have the same net income as last year? A. P1,042,000 C. P1,150,000 B. P1,350,000 D. P1,630,000 (cia) 63. C ? Amount of sales to earn the same net income last year.  Sales in pesos is FC plus profit divided by CMR. The fixed cost is reduced to P316,000 (i.e., P396,000 – P80,000). The income before income tax (IBIT) is the same as last year at P144,000 (i.e., P100,800 / 70%). The new unit variable cost is P24 [i.e., P20 + (10% x P40)], resulting to a new unit contribution margin of P16 (i.e., P40 - P24) or a CMR of 40% ([16 / P40). The sales in pesos shall be determined as: Sales (pesos) = (FC+ IBIT) / CMR = (P316,000 + P144,000) / 40% = P1,150,000 64. Video King Company sells video tapes. The projected after tax net income for the year is P480,000 based on a sales volume of 200,000 units. It sells the tapes at P64 each. The variable cost consists of P40 unit purchase price (bulk orders) and a handling cost of P8 per unit. Annual fixed cost are P2,400,000 and the company’s income tax rate is 35%. An increase of 10 percent in projected unit sales volume for the year would result in an increased after tax income for the year of A. P120,000 C. P208,000 B. P 48,000 D. P 40,000 (rpcpa) 64. C ? The amount of increase in net income given a 10% increase in unit sales.  First, let us get the incremental contribution margin, then deduct the corresponding tax thereof. The unit contribution margin is P16 (i.e., P64 – P48). With a 10% increase in volume the net income after tax shall increase by P208,000 computed as follows: Incremental CM (200,000 x 10% x P16) P320,000

Chapter 4

Marginal costing and CVP analysis

- Incremental tax (35%) Incremental profit after tax

116

112,000 P208,000

Multi-product sales Questions 65 and 66 are based on the following information. The data below pertain to two types of products manufactured by Korn Corporation: Unit sales price Unit variable costs Product Y P 120 P 70 Product Z 500 200 Fixed costs total P300,000 annually. The expected mix in units is 60% for product Y and 40% for product Z. 65. How much is Korn’s breakeven sales in units? A. 857 C. 2,000 B. 1,111 D. 2,459 rpcpa)

(aicpa /

65. C ? Composite BEP in units.  Composite breakeven point (CBEP) in units is total fixed costs over average UCM. The average UCM is determined as follows: Sales Average UCM Mix Ratio UCM Product Y P 50 60% P 30 Product Z 300 40% 120 Total P150 And the CBEP is 2,000 units computed as follows: CBEP (units) = FC / Ave. UCM -= P300,000 / P150 = 2,000 units 66. How much is Korn’s breakeven sales in pesos? A. P300,000 C. P475,000 B. P420,000 D. P544,000 (aicpa) 66. D ? Composite BEP in pesos.   CBEP in pesos is fixed costs over average CMR, and average CMR is average UCM divided average USP. Average USP is P272, determined as follows: Product Y (P120 x 60%) P 72 Product Z (P500 x 40%) 200 Average USP P272 Average CMR is average UCM divided by average USP and is computed at 55.15% (i.e.., P150 / P272). The CBEP in pesos shall be: CBEP (pesos) = FC / Average CMR = P300,000 / 55.15% = P544,000 Alternatively, the composite breakeven point in pesos may also be determined by getting the sum of individual product sales. First, given that the CBEP in units is 2,000 units, the allocated BEP in units shall be 1,200 units for product Y (i.e., 2,000

Chapter 4

Marginal costing and CVP analysis

117

units x 60%) and 800 units for product Z (i.e., 2,000 units x 40%). The CBEP in pesos shall be: Product Y (1,200 units x P120) P144,000 Product Z ( 800 units x P 500) 400,000 Composite BEP P544,000 67. Tomas Company sells products X, Y, and Z. Tomas sells three units of X for each unit of Z, and two units of Y for each unit of X. The contribution margins are P1.00 per unit of X, P1.50 per unit of Y, and P3.00 per unit of Z. Fixed costs are P600,000. How many units of X would Tomas sell at the breakeven point? A. 40,000 C. 360,000 B. 120,000 D. 400,000 (aicpa) 67. B ? The number of units of X to sell at the composite BEP.  First, compute the CBEP in units, then, allocate it among the products. CBEP is fixed costs over average UCM. The average UCM is determined as follows: Sales mix UCM Sales mix ratio Ave. UCM X 3 P1.00 3/10 P0.30 Y (3 x 2) 6 1.50 6/10 0.90 Z 1 3.00 1/10 0.30 10 P1.50 Therefore, the CBEP in units shall be 400,000 units (i.e., P600,000 / P1.50) and the share of product X is 120,000 units (i.e., 400,000 units x 3/10). 68. Maribel is selling three products: Red, White, and Blue. The company sells three units of Red for every unit of Blue, and two units of White for every unit of Red. Fixed costs are P720,000. Contribution margin are: P 1.90 per unit of Red 2.00 per unit of White 2.30 per unit of Blue How many units of White would the company sell at breakeven point? A. 360,000 C. 72,000 B. 108,000 D. 216,000 (rpcpa) 68. D ? The number of units of White to sell at breakeven point.  First, let us determine the composite breakeven point by dividing the fixed costs by the average unit contribution margin (UCM). The average unit contribution margin is calculated by multiplying the individual UCM of the products with their respective sales mix ratio. Now, the sales mix ratio is not readily given, but is to be obtained as follows: If: 3 Reds = 1 Blue Therefore: 2 Whites = 1 Red 6 Whites = 1 Blue And, the sales mix is: Red = 3

Chapter 4

Marginal costing and CVP analysis

118

White = Blue =

6 1 10 The average UCM is determined as: Red P1.90 x 3/10 = P .57 White 2.00 x 6/10 = 1.20 Blue 2.30 x 1/10 = 0.23 Average UCM P2.00 The composite breakeven point (CBEP) is 360,000 units (i.e., P720,000 / P2.00). With the CBEP already determined, the share of each product on the CBEP shall be calculated based on sales mix ratio. The share of product White in the composite breakeven point is 216,000 units (i.e., 360,000 units x 6/10). 69. Laboratorio Unico, Inc. formulates and sells three major chemicals: C1, C2 and C3. It sells to industrial users who use and buy these chemicals in the following ratio: three (3) measures of C1 per one (1) measure of C3, two (2) measures of C2 per one (1) measure of C1. The company makes the following contribution margin per measure: C1 P30 C2 P45 C3 P90 Fixed costs amounted to P1.8 million. At break-even point, the volume of C3 to be sold would be A. 12,000 C. 24,000 B. 36,000 D. 4,000 (rpcpa ) 69. D ? The volume of C3 to be sold at breakeven point.  Composite BEP is fixed cost divide by average UCM. The fixed costs are given. The average UCM is determined by multiplying the individual UCM with their sales mix ratio, as follows: 3 C1 = 1 C3 2 C2 = 1 C1 Therefore: 6 C2 = 1 C3 The sales mix ratio is: C1 3 = 3/10 C2 6 = 6/10 C3 1 = 1/10 10 The average UCM is C1 = P30 x 3/10 = P 9 C2 = 45 x 6/10 = 27 C3 = 90 x 1/10 = 9 Average UCM P45 The composite BEP is 40,000 units (i.e., P1.8 million/P45). The composite BEP should be distributed among the products by their sales mix ratio. Therefore, C3’s share in the composite BEP is 4,000 units (i.e., 40,000 x 1/10).

Chapter 4

Marginal costing and CVP analysis

119

Questions 70 and 71 are based on the based on the following information. A company sells two products X and Y. The sales mix consists of a composite unit of two units of X for every five units of Y (2:5). Fixed costs are P49,500. The unit contribution margins for X and Y are P2.50 and P1.20, respectively. 70. Considering the company as a whole, the number of composite units to break even is A. 31,500 C. 8,250 B. 4,500 D. 9,900 (cia) 70. A ? Composite BEP in units.  Composite BEP in units is fixed costs divided by average UCM. The average UCM is determined as follows: Product X (P2.50 x 2/7) P0.71429 Product Y (P1.20 x 5/7) 0.85714 Average UCM P1.57143 Composite BEP (pesos) = FC / Average UCM = P49,500 / P1.57143 = 31,500 units 71. If the company had an operating income of P22,000,. the unit sales must have been Product X Product Y Product X Product Y A. 5,000 12,500 C. 23,800 59,500 B. 13,000 32,500 D. 28,600 71,500 (cia) 71. B ? Units sold if operating income is P22,000.  Using the average UCM in question 70 as P1.57143, the composite BEP in units shall be: Composite BEP (pesos) = (FC + IBIT) / Average UCM = (P49,500 + P22,000) / P1.57143 = 45,500 units The CBEP shall be allocated based on their sales mix as follows: Product X (45,500 units x 2/7) = 13,000 units Product Y (45,500 units x 5/7) = 32,500 units Questions 72 through 74 are based on the following information. Margarita Manufacturing Company produces two products for which the following data have been tabulated. Fixed manufacturing cost is applied at a rate of P1.00 per machine hour. Per Unit XY-7 BD-4 Selling price P4.00 P3.00 Variable manufacturing costs P2.00 P1.50 Fixed manufacturing cost P .75 P 20 Variable selling costs P1.00 P1.00

Chapter 4

Marginal costing and CVP analysis

120

The sales manager has a P160,000 increase in the money to the most profitable product. The products are not substitutes for one another in the eyes of the company’s customers. 72. Suppose the sales manager chooses to devote the entire P160,000 to increased advertising for XY-7. The minimum increase in sales units of XY-7 to offset the increased advertising is A. 640,000 units. C. 128,000 units. B. 160,000 units. D. 80,000 units. (cma) 72. B ? The increase in unit sales to offset the increase in advertising.  The unit variable cost of XY-7 is P3.00 (i.e., P2 + P1), and its UCM is P1.00 (i.e., P4 – P3). The increased in unit sales to offset the increased in advertising is: Increase in unit sales = Increase in fixed cost / UCM = P160,000 / P1 = 160,000 units 73. Suppose the sales manager chooses to devote the entire P160, 000 to increase advertising for BD-4, the minimum increase in revenues of BD-4 to offset the increased advertising would be A. P160,000 C. P 960, 000 B. P320,000 D. P1,600,000 (cma) 73. C ? The increase in peso sales of produce BD-4 to offset the increase in advertising expense.  The unit variable cost of BD-4 is P2.50 (i.e., P1.50 + P1.00), and its UCM is P0.50 (i.e., P3.00 – P2.50). Its CMR is 16-2/3 %. The increase in peso sales to offset the increase in fixed advertising cost is: Increase in peso sales = Increase in fixed cost / CMR = P160,000 / 16-2/3% = P960,000 74. Suppose Margarita has only 100,000 machine hours that can be made available to produce additional units of XY-7 and BD-4. If the potential increase in sales units for either product resulting from advertising is far in excess of this production capacity, which product should be advertised and what is the estimated increase in contribution margin earned? A. Product XY-7 should be produced, yielding a contribution margin of P75,000. B. Product Xy-7 should be produced, yielding a contribution margin of P133, 333. C. Product BD-4 should be produced, yielding a contribution margin of P187,500. D. Product BD-4 should be produced, yielding a contribution margin of P250, 000.

Chapter 4

Marginal costing and CVP analysis

121 (cma)

74. D ? The product to be sold and its estimated increase in contribution margin.  The unit contribution margin of each product must be converted into contribution margin per hour to determine which is more profitable. It is given that the fixed overhead is applied at P1.00 per hour. The contribution margin per hour shall be computed as follows: XY-7 BD-4 UCM (as computed in the preceding number) P1.00 P0.50 ÷ No, of hours per unit (P.75 per unit / P1 per hour) 0.75 hr. (P.20 per unit / P1 per hour) 0.20 hr. Contribution margin per hour P1.33 P2.50 Product BD-4 gives a higher CM per hour and should be prioritized for production and sales. The total CM in producing BD-4 is P250,000 (i.e., 100,000 hours x P2.50 per hour). 75. In a multi-product company, as the mix of the products being sold changes, the overall contribution margin ratio will also change. If the shift in mix is toward the less profitable products, then the contribution margin ratio will A. Fall C. Not change B. Rise D. Change in direct proportion to break-even point (rpcpa) 75. A ? The effect to contribution margin ratio (CMR) if the sales mix is shifted towards the less profitable product.  A shift in mix towards the less profitable product will make the average unit contribution margin to decline. This will result to a lower CMR (choice-letter “a” is correct). Choice-letter “b” is incorrect because it contradicts the correct analysis. Choiceletter “c” is also incorrect because CMR is affected by the change in sales mix. Choice-letter “d” is incorrect because the change in BEP is not directly proportional to the change in CMR. 76. Sari- sari Corporation is a multiple-product firm. In their review of operations, they decided to shift the sales mix from less profitable products to more profitable products, accounting for 30% of gross sales. This will cause the company’s breakeven profit to A. Decrease C. Increase B. Change by 15%. D. Not change (rpcpa) 76. A ? The effect on the breakeven point (BEP) if the sales mix is shifted from less profitable products to more profitable products.  The shift in sales mix from the less profitable to the more profitable product would increase the average contribution margin. This means a decrease in the composite

Chapter 4

Marginal costing and CVP analysis

122

BEP (choice-letter “a” is correct). Choice-letter “b” is not determinable and is incorrect. Choice-letter “c” is incorrect because it contradicts the results of the correct analysis. Choice-letter “d” is incorrect because there is a change in the breakeven point due to the change in sales mix. 77. Phil. Frames Inc. has the following revenue and cost budgets for the two product it sells Plastic Frames

Sales price Direct materials Direct labor Fixed overhead Net income per unit Budgeted units sales

P50.00 (10.00) (15.00) (15.00) P10.00 100,000

Glass Frames

P75.00 (15.00) (25.00) (20.00) P15.00 300,000

The budgeted unit sales equal the current unit demand, and total fixed overhead for the year in budgeted at P4,875,000. Assume that the company plans to maintain the proportional mix. In numerical calculations, the company rounds to the nearest centavo and unit. The total number of units the company needs to produce and sell to break-even is A. 300,000 units C. 150,000 units B. 354,545 units D. 75,000 units (rpcpa) 77. C ? The composite breakeven point.  Composite BEP refers to the point where the total sales of all the products made by the company is equal to the total costs. These products made by the company is equal to the total process, and costs structure. The composite BEP is equal to fixed costs divide by average unit contribution margin (UCM). The fixed costs is given at P4,875,000. The average UCM shall be computed using the sales mix ratio of 1:3 for plastic and glass, respectively, based on the given budgeted unit sales. Plastic Glass Average UCM Unit sales price P 50.00 P 75.00 Unit direct materials cost ( 10,.00) ( 15.00) Unit direct labor cost ( 15.00) ( 25.00) Unit contribution margin P 25.00 P 35.00 X Sales mix ratio (1:3) ¼ ¾ Adjusted UCM P 6.25 P 26.25 P32.50 Composite BEP (units)

= Fixed costs / Average UCM = P4,875,000 / P32.50 = 150,000 units

78. The following revenues and cost budget for two products Things Inc. sells are made available: Plastic Things Glass Things Sales price P50.000 P75.00 Direct materials (10.00) (15.00) Direct labor (15.00) (25.00)

Chapter 4 Fixed overhead Net income per unit Budgeted units sales

Marginal costing and CVP analysis (15.00) P10.00 150,000

123

(20.00) P15.00 300,000

The budgeted unit sales equal the current unit demand, and total fixed overhead for the year is budgeted at P4,875,000. Assume that the company plans to maintain the same proportional mix. In numerical calculations, the company rounds to the nearest centavos and unit. The total number of units Thing Inc. needs to produce and sell to break-even is A. 102,632 units. C. 171,958 units. B. 153,947 units. D. 418,455 units. (rpcpa) 78. B ? The composite breakeven point.  Composite breakeven point is the total or combined sales in units of all products produced and processed in the same plant in order to breakeven (e.g., total sales equal total costs.) The composite BEP is computed by dividing the total fixed costs and expenses by the average unit contribution margin (UCM). The average UCM is determined below: Plastic Glass Average UCM Unit sales price P50.00 P75.00 Unit variable costs (25.00) (40.00) Unit contribution margin 25.00 35.00 X Sales mix ratio (15:30) 15/45 30/45 Adjusted UCM P 8.33 P23.33 P31.66 Composite BEP (units) = P4,875,000 / P 31.66 = 153,947 units The sales mix ratio (i.e., 15:30) is taken from the relationship of the budgeted units of 150,000 units and 300,000 units for plastic and glass products, respectively.

A. B. C. D.

Breakeven and CVP graphs 79. In a profit-volume graph, the cost/volume/profit relationships are represented. The vertical axis is the profit in pesos and the horizontal axis is the volume in units. The diagonal line is the contribution margin line. The point at which the contribution margin line intersects the zero profit line is the point: At which the volume level is zero. At which the total costs equal the total sales. At which sale increases. At which total variable costs equal total sales. (rpcpa) 79. B ? The point at which the contribution margin line intersects the zero profit line in the profit-volume graph.  If the contribution margin line intersects the zero profit point, then there is no profit and the business operates at the breakeven point. At this point, total costs equal the total sales. Choice-letter “b” is correct. Choice-letter “a” is incorrect because there is a sale made and therefore the volume level is higher than zero. Choice-letter “c” is incorrect because a change in sale would affect the amount of contribution margin and not at point zero.. Choice-

Chapter 4

Marginal costing and CVP analysis

124

letter “d” is also incorrect because if total variable costs and expenses equal total sales, then there is no contribution margin. 80. When using the graph method, if unit output exceeds the break-even point, A. Expenses are extremely high relative to revenues. B. There is loss because the total cost line exceeds the total revenue line. C. Total sales exceed total cost. D. There is profit since the total cost line exceeds the total revenue line. (rpcpa) 80. C ? Using the graph method, the event that will happen when unit output exceeds the breakeven point.  After the breakeven point, there is already a margin of safety and profit because total sales already exceed total costs and total contribution margin is greater than fixed cost and expenses (choice-letter “c” is correct). Choice-letters “a” and “b” are incorrect because after breakeven point revenues are already higher than expenses, and therefore, will result to profit. Choice-letter “d” is incorrect because if there is profit, the total cost line should be exceeded by, and not to exceed, the total revenue line. 81. The most important use of the cost-volume-profit graph is to show A. The breakeven point. B. The cost/margin ratio at various levels of sale activity. C. The relationships among volume, cost, revenues, over wide ranges of activity. D. The determination of cross over point. (rpcpa) 81. C ? The most important use of the CVP graph.  The CVP graph and even the BEP graph show the relationships among revenues, costs and volume within a given relevant range. The breakeven point is not the main focus of presentation but its focus is the effects and relationships to profit of revenues, costs and volume. Hence, choice-letter “a” is incorrect. The contribution margin ratio is assumed to be constant over the relevant range, choice-letter “b” is incorrect. Choice-letter “d” is an irrelevant choice. Sensitivity analysis Questions 82 through 84 are based on the following in formation. MultiFrame Company has the following revenue and cost budgets for the two products it sells: Plastic Frames Glass Frames Sales price P10.00 P 15.00 Direct materials (2.00) (3.00) Direct Labor (3.00) (5.00) Fixed overhead (3.00) (2.75) Net income per unit P 2.00 P 4.25 Budgeted unit sales 100,000 300,000

Chapter 4

Marginal costing and CVP analysis

125

The budgeted unit sales equal the current unit demand, and total fixed overhead for the year is budgeted at P975,000. Assume that the company plans to maintain the same proportional mix. In numerical calculation, MultiFrame rounds to the nearest cent and unit 82. The total number of units MultiFrame needs to produce and sell to break even is A. 150,000 units. C. 100,000 units. B. 153,947 units. D. 300,000 units. (cma) 82. A ? The total number of units needed to produce and sell to breakeven.   The requirement refers to the composite breakeven point. It is calculated as fixed costs over average UCM. The sales mix shall be based on budgeted sales (1:3) and the sales mix ratios are ¼ and ¾ for Plastic and Glass, respectively. The average UCM is determined as follows:

Plastic (P10 – P5) Glass (P15 – P8) Total

Sales UCM mix ratio P5.00 ¼ 7.00 ¾

Ave. UCM P1.25 5.25 P 6.50

The composite BEP shall be 150,000 units (P975,000 / P6.50). 83. The total number of units needed to break even If the budgeted direct labor cost were P2 for plastic frames instead of P3 is A. 144,444 units. C. 153,947 units. B. 150,000 units. D. 100,000 units. (cma) 83. A ? The new composite BEP assuming a change in direct labor cost.  The direct labor cost of plastic frames decreases to P2 from a previous balance of P3 or a decrease of P1. This decrease in unit variable cost will increase the UCM of plastic frames to P6 (i.e., P5 + P1). With this change, the new average UCM shall be: Plastic frames (P6 x ¼) P1.50 Glass frames (P7 x ¾) 5.25 Average UCM P6.75 The composite BEP shall be 144,444 units (i.e., P975,000 / P6.75). 84. The total number of units needed to break even if sales were budgeted at 150,000 units of plastic frames and 300,000 units of glass frames with all other costs remaining constant is A. 144,444 units. C. 153,947 units.

Chapter 4 B.

Marginal costing and CVP analysis 150,000 units. (cma)

126

D. 450,000 units.

84. C ? New composite BEP given a change in sales mix.   The UCM of plastic frames and glass remains constant at P5 for plastic frames and P7 for glass frames. The new sales mix is changed to 15:30 for plastic frames and glass frame, respectively. The new average UCM shall be: Plastic frames (P5 x 15/45) P1.67 Glass frames (P7 x 30/45) 4.67 Average UCM P6.34 or P6.33333 The new composite BEP shall be 153,947 units (i.e., P975,000 ÷ P6.333333). 85. Calculate the overall breakeven point in terms of units if the company believes that the current price of P40 is too high and the firm faces stiff competition. After all the sensitivity analysis is done, it was decided by the management committee to lower the price to P36 without sacrificing the quality of the product. What is the new breakeven point if fixed costs are P309,750 and unit contribution margin is P6? A. 51,625 C. 31,250 B. 39,125 D. 43,750 (rpcpa) 85. A ? The new breakeven point in units.  The new breakeven point is equal to new fixed costs divided by the new unit contribution margin. Applying this, we will have a new BEP in units of 51, 625 units (i.e., P309,750 / P6). Questions 86 and 87 are based on the following information. Presented below are the results of operations of Softtouch Products, Inc., for 2005:

Sales (150,000 units) Cost of goods sold: Fixed Variable Gross profit Selling and administrative: Fixed Variable Income before taxes

P 600,000 P 150,000 300,000 39,000 45,000

450,000 150,000 84,000 P 66,000

The company is concerned about the expected increase in fixed manufacturing costs by 50% if it will buy a new equipment with a higher production capacity. However, study shows that with the use of the new equipment sales volume in units are expected to increase by 40% while variable manufacturing costs will decrease from P2.00 to P1.50 per unit. The total fixed selling and administrative expenses and variable selling and administrative expenses will remain the same. The selling price per unit will also remain the same. The company has been operating at full capacity. If the company will buy the new equipment,

Chapter 4

Marginal costing and CVP analysis

127

86. What would be the breakeven point in terms of units? A. 128,000 C. 176,000 B. 66,000 D. 105,000 (rpcpa) 86. A ? The new breakeven point if the company buys the new equipment.  The changes in the data could be analyzed as follows: Before After Unit sales price (P600,000 / 150,000) P 4.00 P 4.00 Unit variable production costs (P300,000 / 150,000) ( 2.00) ( 1.50) Unit variable expenses (P45,000 / 150,000) ( 0.30) ( 0.30) Unit contribution margin P 1.70 P 2.20 Total fixed manufacturing cost P150,000 P225,000 (P150,000x150%) Total fixed expenses 39,000 39,000 Total fixed costs and expenses P189,000 P264,000 The new breakeven point in units is (P264,000 / P2.20 120,000 87. What is the maximum expected income before income tax? A. P198,000 C. P306,000 B. P 216,000 D. P 288,000 (rpcpa) 87. A ? The maximum expected income before income tax.  Income before income tax is contribution margin less fixed costs and expenses. Therefore, the maximum income before income tax is P198,000, calculated as: Contribution margin (150,000 units x 140% x P2.20) P462,000 - Fixed costs and expenses 264,000 Income before income tax P198,000 Questions 88 and 99 are based on the following information. The Kabayan Company manufactures and sells Batik handbags to assorted prints. Data for the previous year were as follows: Selling price per piece P 8.00 Variable cost per piece P 2.00 No. of pieces to breakeven 25,000 Net income last year P5,850 For the coming year, the company estimates that the selling price will be P9.50 per piece, Variable cost to manufacture will increase by 25%, and fixed costs will increase by 10%. Income tax rate of 35% will not change. 88. What is the selling price per piece that would give the same contribution margin rate as previous year? A. P10.00 C. P 8.00 B. P 9.50 D. P10.50 (rpcpa)

Chapter 4

Marginal costing and CVP analysis

128

88. A ? The selling price per unit that would give the same CMR as the previous year.  The old variable cost ratio is 25% (i.e. P2 / P8) and the CMRatio is 75% (i.e., 100% – 25%). If the new unit variable cost is P2.50 (i.e., P2.00 x 125%), then the new unit sales price to maintain the same CMR of 75% and the same VCRatio of 25% shall be P10.00 (i.e., P2.50 / 25%). 89. If sales for the coming year are expected to exceed last year’s by 1,800 pieces, what would be the expected sales volume for the coming year? A. 28,300 C. 26,800 B. 27,775 D. 26,500 . (rpcpa) 89. A ? The expected sales volume in the coming year.  The sales volume in the coming year shall be the sum of the sales volume last year plus the expected increase of 1,800 units. The sales volume last year and the new sales volume is determined as follows: Fixed costs (25,000 x P6) P150,000 Income before income tax (P5,850 / 65%) 9,000 Contribution margin 159,000 / Contribution margin ratio 75 % Sales 212,000 / Unit sales price P 8.00 Sales volume last year 26,500 units Add: Increase in sales volume this year 1,800 Expected sales in units in the coming year 28,300 units 90. Wilson Company prepared the following preliminary forecast concerning product G for 2006 assuming no expenditure for advertising: Selling price per unit P 10 Units sales 100,000 Variable costs P 600,000 Fixed costs P 300,000 Based on market study in December 2005, Wilson estimated that it could increase the unit selling price by 15% and increase the unit sales volume by 10% if P100,000 were spent on advertising. Assuming that Wilson incorporates these changes in its 2006 forecast, what should be the operating income from product G? A. P 175,000 C. P 205,000 B. P 190,000 D. P 365,000 (aicpa) 90. C ? Operating income given some changes in the variables of profit.  Profit is sales less costs and expenses. The unit variable cost is P6 (i.e, P600,000 / 100,000 units). The operating income in 2006 is calculated as follows: Sales [(100,000 units x 110%) x (P10 x 115%)] P1,265,000

Chapter 4

Marginal costing and CVP analysis

- Variable costs [(100,000 units x 110%] x P6] Contribution margin - Fixed costs (P300,000 + P100,000) Operating income

129

660,000 605,000 400,000 P 205,000

91. When used in cost-volume-profit analysis, sensitivity analysis A. Determines the most profitable mix of product to be sold. B. Allows the decision maker to introduce probabilities in the evaluation of decision alternatives. C. Computes profit per unit of production and determines the optimum production of the company. D. Is done through various possible scenarios and computes the impact on profit of various predictions of future events. (rpcpa) 91. D ? The statement describing sensitivity analysis as used in cost-volume-profit analysis.  Sensitivity analysis (or simulation analysis) is the process of determining the effects of the possible changes in any of the variables to the overall outcome in a given circumstance. Applied in the CVP analysis, it refers to the process of determining the effects to profits of the possible changes in the variables affecting profit (choice-letter “d” is correct). Choice-letter “a” is incorrect because it refers to linear programming where the optimum mix of production and sales is computed to maximize profit. Choice-letter “b” is incorrect because it refers to making decisions under uncertainty where the concept of probabilities is applied. Choice-letter “c” is incorrect because it also refers to linear programming. 92. Rainbow Company’s controller developed the following variable-costing income statement for 2006: Per Unit Revenues (150,000 units at P30) P4,500,000 P30 Variable costs: Direct materials P1,050,000 7 Direct labor 1,500,000 10 Mfg. Overhead 300,000 2 Selling & marketing 300,000 2 ( 3,150,000) 21 Contribution margin 1,350,000 9 Fixed costs: Mfg. overhead P 600,000 4 Selling & marketing 300,000 2 ( 900,000) 6 Operating income P 450,000 P3 Rainbow Company based its 2006 budget on the assumption that fixed costs, unit sales, and the sales price would remain as they were in 2005, but with operating income being reduced to P300, 000. By July of 2006, the controller was able to

Chapter 4

Marginal costing and CVP analysis

130

predict that unit sales would increase over 2005 levels by 10%. Based on the 2006 budget and the new information, the predicted 2006 operating income would be A. P300,000 C. P420,000 B. P330,000 D. P585,000 (cia) 92. C ? The predicted operating income in 2006.  Operating income is sales less total costs and expenses. It is predicted that total fixed costs and unit sales price are unchanged, units sold increases by 10%, and profit decreases to P300,000, if units sold remain constant. The UCM as predicted on January 1, 2006 is: Projected operating income P 300,000 + Fixed costs 900,000 Projected CM 1,200,000 / Units sold (assumed unchanged) 150,000 units UCM if units were unchanged P 8.00 The projected operating income is revised on July 1, 2006 when volume is expected to increase by 10% over that of the 2005 level, as follows: CM (150,000 x 110% x P8) P1,320,000 - Fixed costs 900,000 Projected operating income P 420,000 Questions 93 and 94 are based on the following information. Levi Company has developed a new project that will be marketed for the first time during the next fiscal year. Although the Marketing Department estimates that 35,000 units could be sold at P36 per unit, Levi management has allocated only enough manufacturing capacity to produce a maximum of 25,000 units of the new product annually. The fixed costs associated with the new product are budgeted at P450,000 for the year, which includes P60, 000 for depreciation on new manufacturing equipment. Data associated with each unit of product are presented below. Levi is subject to a 40% income tax rate. Variable Cost Direct material P 7.00 Direct labor 3.50 Manufacturing overhead 4.00 Total variable manufacturing cost 14.50 Selling expenses 1.50 Total variable cost P16.00 93. The maximum after-tax income that can be earned by Levi Company from sales of the new product during the next fiscal year is A. P30,000 C. P110,000 B. P50,000 D. P 66,000 (cma) 93. A

Chapter 4

Marginal costing and CVP analysis

131

? The maximum after-tax income of the new product next year.  The management has decided to produce and sell only 25,000 units. The UCM of the product is P20 (i.e., P36 – P16). The net income is computed as follows: Contribution margin (25,000 units x P20) P500,000 - Fixed costs 450,000 Income before income tax 50,000 - Income tax (40%) 20,000 Net income P 30,000 94. Levi Company’s management has stipulated that it will not approve the continued manufacture of the new product after the nest fiscal year unless the after-tax income is at least P75, 000 the first year. The unit-selling price to achieve this target income must be at least. A. P37.00 C. P34.60 B. P36.60 D. P39.00 (cma) 94. D ? The targeted unit sales price.   Unit sales price is unit variable cost plus unit contribution margin. computed below: IBIT [P75,000 / (1-.40)] P125,000 Fixed costs 450,000 Contribution margin P575,000 Unit contribution margin (P575,000 / 25,000 units) P Unit variable cost Unit sales price P

The UCM is

23.00 16.00 39.00

95. During 2005, David Lab supplied hospital with an comprehensive diagnostic kit for P120. At a volume of 80,000 kits, David had fixed costs of P 1 million and operating income before income taxes of P200,000. Because of an adverse legal decision, David’s 2006 liability insurance increased by P1.2 million over 2005. Assuming the volume and other costs are unchanged, what should the 2006 price be if Thor is to make the same P200,000 operating income before income taxes? A. P 120.00 C. P 150.00 B. P 135.00 D. P 240.00 (aicpa) 95. B ? Unit sales price if operating income is P200,000.   Unit sales price is unit variable cost plus unit contribution margin. The unit variable cost last year does not change, while total fixed costs increases to P2.2 million. The UCM in 2006 shall be determined as follows: 2005 2006 Income before income tax P 200,000 P 200,000 Fixed costs 1,000,000 2,200,000 (P1 million + P1.2 million) Contribution margin

1,200,000

2,400,000

Chapter 4

Marginal costing and CVP analysis

/ Units sold in 2004 80,000 Unit contribution margin P 15 Unit variable cost (P120 – P15) P 105 Unit sales price

P P P

132

80,000 30 105 135

Questions 96 and 97 are based on the following information. A wholesale distributing company has budgeted its before-tax profit to be P643,500 for 2005. The company is preparing its annual budget for 2006 and has accumulated the following data: Projected annual revenues P6,000,000 Variable costs as a percent of revenues: Cost of merchandise 30% Sales commissions 5% Shipping expenses 10% Annual fixed operating costs: Selling expenses P 772,200 Administrative expenses P1,801,800 96. If the wholesale distributing company wants to earn the same before-tax operating income in 2006 as budgeted for 2005, the annual revenues would not be the projected P6 million but would have to A. P 4,950,000 C. P 5,850,000 B. P 5,362,000 D. P 7,150,000 (cia) 96. C ? Projected sales in 2006.  The variable cost ratio is 45% (i.e., 30% + 5% + 10%) and CMR is automatically 55%. Total fixed costs is P2,574,000 (i.e., P772,200 + P1,801,800). If the income in 2005 is retained in 2006, the needed sales in 2005 shall be P5,850,000 calculated as follows: Sales (pesos) = (FC + IBIT) / CMR = (P2,574,000 + P643,500) / 55% = P5,850,000 97. Using the original P 6 million projection, the wholesale distributing company’s margin of safety in terms of revenues for 2006 would be A. P 82,500 C. P 280,000 B. P 150,000 D. P1,320,000 (cia) 97. D ? Margin of safety in 2006.  Margin of safety is budgeted sales less breakeven sales, computed as follows: Budgeted sales P6,000,000 - Breakeven sales (P2,574,000 / 55%) 4,680,000 Margin of safety P1,320,000 Operating leverage 98. Bi Corporation is operationally, a highly leveraged company, that is, it has high fixed costs and low variable costs. As such, small changes in sales volume result in A. Proportionate change in net income.

Chapter 4 B. C. D.

Marginal costing and CVP analysis

Large changes in net income. Negligible change in net income. No change in net income.

133

(rpcpa)

98. B ? Effect of a high degree of operating leverage if there are small changes in sales volume.  A company with a high degree of operating leverage would also result to a high degree of change in EBIT (earnings before income tax) in every small change in net sales. This relationship is expressed in the formula: ٪Δ in EBIT = ٪Δ in Net Sales x Degree of Operating Leverage Choice-letter “a” is incorrect because the percentage change in net income is not only affected by the degree of operating leverage but that of the percentage change in net sales. Therefore, a change in sales volume does not result to a proportionate change in net income. Choice-letters “c” and “d” are also incorrect because the percentage change in net income is directly affected by the degree of operating leverage. 99. The percentage change in earning before interest and taxes associated with the percentage change in revenues is the degree of A. Operating leverage. C. Breakeven leverage. B. Financial leverage. D. Combined leverage. (cia) 99. A ? Percentage change in EBIT with the percentage change in revenues  The percentage change in EBIT in relation with the percentage change in sales measures the elasticity of change in profit with the change in sales. This is called the degree of operating leverage (DOL). Another mode of determining the DOL is dividing contribution margin by the operating income. The higher the DOL, the higher the change in EBIT in times of increasing trend in sales. The DOL computations are presented below: That is, DOL = CM / EBIT Also, DOL = Percentage change in EBIT x Percentage change in sales Indifference point 100.Two companies are expected to have annual sales of 1 million decks of playing cards next year. Estimates for next year are presented below: Company 1 Company 2 Selling price per deck P3.00 P3.00 Cost of paper per deck .62 .65 Printing ink per deck .13 .15 Labor per deck .75 1.25 Variable overhead per deck .30 .35 Fixed costs P960,000 P252,000 Given these data, which of the following responses is correct? Volume in Units

Volume in Units

Chapter 4

Marginal costing and CVP analysis

Breakeven Breakeven at which profits Points in Point in of Company 1 Units for Units for and Company 2 Company 1 Company 2 Are Equal A. 800,000 420,000 1,180.000 1,000.000 B. 800,000 420,000 1,000.000 1,180.000

134

Breakeven Breakeven at which profits Point in Point in of Company 1 Units for Units for and Company 2 Company 1 Company 2 Are Equal C. 533.334 105,000 D. 533,334

105,000 (cia)

100.A ? BEP and indifference point.  The BEP for each company shall be: Company 1 Company 2 Unit sales price P 3.00 P 3.00 Unit variable cost (P.62 + P.13 + P.75 + P.30) 1.80 2.40 (P.65 + P.15 + P1.25 + P.35) Unit contribution margin 1.20 0.60 BEP in units (P960,000 ÷ P1.20) 800,00 420,000 (P252,000 ÷ P.60) The indifference point is where the results or profits between the alternatives would be the same. The profits of the two companies are expressed as follows (assume “x” as the number of units sold): P1 = 1.20x + P960,000 where: P1 = profit of Co. 1 P2 = 0.60x – P252,000 P2 = profit of Co.2 At indifference point: Therefore:

P1 = P2 1.20x – P960, 000 = 0.60x – P252,000 x = 1,180,000 units

Miscellaneous 101.NCB Inc. manufactures computers tables. It has an investment of P1,750,000 in assets and expects a 25% return on investment. Its fixed production cost for 2,000 units is P550,000 plus an additional P150,000 for selling and administrative expenses. The variable cost to manufacture is P1,500 per table. The selling price per table should be: A. P2,068.75 C. P2,531.25 B. P1,850.00 D. P2,725.00 (rpcpa) 101.A ? The selling price per table.  Unit sales price is the sum of unit variable costs and expenses, unit fixed costs and expenses, and unit profit margin as follows: Unit variable costs P1,500.00 Unit fixed cost P550,000/2,000) 275.00 Unit fixed expenses (P150,000/2,000) 75.00 Unit profit margin [(P1,750,000 x 25%) / 2,000 units] 218.75 Unit sales price P2,068.75

Chapter 4

Marginal costing and CVP analysis

135

102.Using absorption costing, the determination of the break-even point depends on all of the following, except A. The budgeted level of production B. Achieving targeted production levels C. The number of units sold during the period D. The level of fixed manufacturing overhead. (rpcpa) 102.B ? The item that is not included in determining the breakeven point.  Choice-letter “b”, achieving targeted production levels, does not affect BEP calculation and is the correct answer. Choice-letter “a”, budgeted level of production, is the basis in computing total fixed costs and unit variable costs, and is relevant in determining the breakeven point. Choice-letter “c”, the number of units sold, is also important in determining the contribution margin and the BEP. Choice-letter “d”, the level of fixed manufacturing overhead, is definitely relevant in determining the BEP. 103.Hello Garci Company sells 50,000 units of “yo” a top-of-the-line garden srinkle. These were taken from the company’s records: • Accounts receivable P129,000. • Days of sales outstanding 15 days. • Contribution margin ratio 49%. • Profit for the period was P485,000. The ending receivables balance is the average balance during the year. Assume a 360-day year. All sales are on credit. Determine the company’s breakeven revenue. A. P2,106,122 C. P2,023,608 B. P1,032,040 D. P3,096,000 103.A ? Breakeven revenue.  Breakeven revenue or breakeven peso sales is equal to fixed cost over CMRatio. The fixed cost is not given but could be determined by getting the difference of contribution margin and profit before tax. The amount of contribution margin is equal to sales multiplied by the CMRatio. The CMR is given but the amount of net sales is to be computed using the receivable turnover analysis. Net sales equal average accounts receivable balance times the receivable turnover. The A/R turnover is equal to 360 days in a year divided by the collection period. The procedural analysis follows: a. Average Accounts Receivable P 129,000 x A/Receivable Turnover (360 days/15 days) 24 times Net sales 3,096,000 X CMRatio 49% Contribution margin 1,517,040 Less: Profit before tax 485,000 Fixed costs P1,032,040 b. BEP (pesos) = P1,032,040 / 49%

=

P2,106,204

Chapter 4

Marginal costing and CVP analysis

136

104.State whether the following statements are true or false. Statement 1-The breakeven point is defined as the sum of variable expenses and fixed expenses. Statement 2-As sales exceed the breakeven point, a low contribution margin percentage would result in lower profit than would a high contribution margin percentages. Statement 3-All fixed costs are treated as period costs when variable costing is used. Statement 1 Statement 2 Statement 3

A. B.

True False

True True

True True

Statement 1 Statement 2 Statement 3

C. D.

False False

False False

True False (rpcpa)

104.B ? Identify the statements as true or false.  Statement 1 is false because the sum of variable expenses and fixed expenses is the total expenses, and not the breakeven point. Statement 2 is correct because a higher contribution margin ratio would result to higher profit for every peso of sale after the breakeven sales. Statement 3 is also true because fixed costs and expenses are considered as period costs (automatically deducted from revenues) under the variable costing system. Hence, choice-letter “b “ is the correct answer.

 done 

View more...

Comments

Copyright ©2017 KUPDF Inc.
SUPPORT KUPDF